Asymmetric Ciphers Questions and Answers – Knapsack/ Merkle – Hellman/ RSA Cryptosystem – I

This set of Cryptography Multiple Choice Questions & Answers (MCQs) focuses on “Knapsack/ Merkle – Hellman/ RSA Cryptosystem”.

1. Imagine you had a set of weights {62, 93, 26, 52, 166, 48, 91, and 141}. Find subset that sums to V = 302.
a) {62, 48, 166, 52}
b) {141, 26, 52, 48}
c) {93, 26, 91, 48}
d) {62, 26, 166, 48}
View Answer

Answer: d
Explanation: {62, 26, 166, 48} =302.

2. For the Knapsack: {1 6 8 15 24}, Find the cipher text value for the plain text 10011.
a) 40
b) 22
c) 31
d) 47
View Answer

Answer: a
Explanation: 1+15+24 = 40.

3. For the Knapsack: {1 6 8 15 24}, find the plain text code if the ciphertext is 38.
a) 10010
b) 01101
c) 01001
d) 01110
View Answer

Answer: b
Explanation: If someone sends you the code 38 this can only have come from the plain text 01101.
advertisement
advertisement

4. Set {1, 2, 3, 9, 10, and 24} is superincreasing.
a) True
b) False
View Answer

Answer: b
Explanation: It is not because 10 < 1+2+3+9.

5. A superincreasing knapsack problem is ____ to solve than a jumbled knapsack.
a) Easier
b) Tougher
c) Shorter
d) Lengthier
View Answer

Answer: a
Explanation: A superincreasing knapsack is chosen to make computations easier while manual calculations of knapsack problems.
Sanfoundry Certification Contest of the Month is Live. 100+ Subjects. Participate Now!

6. Consider knapsack that weighs 23 that has been made from the weights of the superincreasing series {1, 2, 4, 9, 20, and 38}. Find the ‘n’.
a) 011111
b) 010011
c) 010111
d) 010010
View Answer

Answer: b
Explanation: v0=1, v1=2, v2=4, v3=9, v4=20, v5=38
K=6, V=23
Starting from largest number:
v5 > V then ϵ_5=0
v4 < V then V = V – v4 = 23 – 20 = 3 ϵ_4=1
v3 > V then ϵ_3=0
v2> V then ϵ_2=0
v1 < V then V = V – v1= 3 – 2 = 1 ϵ_1=1
v0 =1 then V = V – v0= 1 – 1 = 0 ϵ_0=1
n= ϵ_5 ϵ_4 ϵ_3 ϵ_2 ϵ_1 ϵ_0 = 010011.

7. Another name for Merkle-Hellman Cryptosystem is
a) RC4
b) Knapsack
c) Rijndael
d) Diffie-Hellman
View Answer

Answer: b
Explanation: Knapsack is another name for Merkel-Hellman Cryptosystem.
advertisement

8. In Merkle-Hellman Cryptosystem, the hard knapsack becomes the private key and the easy knapsack becomes the public key.
a) True
b) False
View Answer

Answer: b
Explanation: The hard knapsack becomes the public key and the easy knapsack becomes the private key.

9. In Merkle-Hellman Cryptosystem, the public key can be used to decrypt messages, but cannot be used to decrypt messages. The private key encrypts the messages.
a) True
b) False
View Answer

Answer: b
Explanation: The public key can be used to encrypt messages, but cannot be used to decrypt messages. The private key decrypts the messages.
advertisement

10. The plaintext message consist of single letters with 5-bit numerical equivalents from (00000)2 to (11001)2. The secret deciphering key is the superincreasing 5-tuple (2, 3, 7, 15, 31), m = 61 and a = 17. Find the ciphertext for the message “WHY”.
a) C= (148, 143, 50)
b) C= (148, 143, 56)
c) C= (143, 148, 92)
d) C= (148, 132,92)
View Answer

Answer: a
Explanation: {wi }= {a vi mod m}
{wi} = { 17×2 mod 61, 17×3 mod 61, 17×7 mod 61, 17×15 mod 61, 17×31 mod 61}
{wi} = {34, 51, 58, 11, and 39}
PlainText In binary Ci
W- 22 10110 148
H – 7 00111 143
Y – 24 11000 50
So that the ciphertext sent will be C= (148, 143, 50).

11. For p = 11 and q = 17 and choose e=7. Apply RSA algorithm where PT message=88 and thus find the CT.
a) 23
b) 64
c) 11
d) 54
View Answer

Answer: c
Explanation: n = pq = 11 × 19 = 187.
C=Me mod n ; C=887 mod 187 ; C = 11 mod 187.

12. For p = 11 and q = 17 and choose e=7. Apply RSA algorithm where Cipher message=11 and thus find the plain text.
a) 88
b) 122
c) 143
d) 111
View Answer

Answer: a
Explanation: n = pq = 11 × 19 = 187.
C=Me mod n ; C=1123 mod 187 ; C = 88 mod 187.

13. In an RSA system the public key of a given user is e = 31, n = 3599. What is the private key of this user?
a) 3031
b) 2412
c) 2432
d) 1023
View Answer

Answer: a
Explanation: By trail and error, we determine that p = 59 and q = 61. Hence f(n) = 58 x 60 = 3480.
Then, using the extended Euclidean algorithm, we find that the multiplicative
inverse of 31 modulo f(n) is 3031.

14. Compute private key (d, p, q) given public key (e=23, n=233 ´ 241=56,153).
a) 35212
b) 12543
c) 19367
d) 32432
View Answer

Answer: c
Explanation: Since n=233 ´ 241=56,153, p=233 and q=241
f(n) = (p – 1)(q – 1) = 55,680
Using Extended Euclidean algorithm, we obtain
d = 23–1 mod 55680 = 19,367.

Sanfoundry Global Education & Learning Series – Cryptography and Network Security.

To practice all areas of Cryptography and Network Security, here is complete set of 1000+ Multiple Choice Questions and Answers.

If you find a mistake in question / option / answer, kindly take a screenshot and email to [email protected]

advertisement
advertisement
Subscribe to our Newsletters (Subject-wise). Participate in the Sanfoundry Certification contest to get free Certificate of Merit. Join our social networks below and stay updated with latest contests, videos, internships and jobs!

Youtube | Telegram | LinkedIn | Instagram | Facebook | Twitter | Pinterest
Manish Bhojasia - Founder & CTO at Sanfoundry
Manish Bhojasia, a technology veteran with 20+ years @ Cisco & Wipro, is Founder and CTO at Sanfoundry. He lives in Bangalore, and focuses on development of Linux Kernel, SAN Technologies, Advanced C, Data Structures & Alogrithms. Stay connected with him at LinkedIn.

Subscribe to his free Masterclasses at Youtube & discussions at Telegram SanfoundryClasses.